How can the recurrence formula for a sequence be found?

  • Thread starter Thread starter Mathematicsss
  • Start date Start date
  • Tags Tags
    Formula Recurrence
Click For Summary
To find a recurrence formula for the sequence defined as (ai) = 1, sqrt(3), sqrt(1+sqrt(3)), and sqrt(1+sqrt(1+sqrt(3))), one should first organize the sequence elements clearly. The sequence can be expressed as a0 = 1, a1 = sqrt(3), a2 = sqrt(1 + a1), and a3 = sqrt(1 + a2). By identifying the relationship between consecutive terms, one can derive a formula where each term is defined in terms of the previous term, specifically a_n = sqrt(1 + a_{n-1}). This approach simplifies the problem and aligns with techniques used in analyzing nested radicals.
Mathematicsss

Homework Statement


Find a recurrence formula for the sequence (ai) = 1, sqrt3, sqrt(1+sqrt3), sqrt(1+sqrt(1+sqrt2)) in terms of i and ai

Homework Equations

The Attempt at a Solution


no idea where to start, this is a bonus question, and I have learned how to solve these type of problems
 
Physics news on Phys.org
I'd investigate nested radicals as this was a favorite of Ramanujan.

As a hint,## x = \sqrt{1 + \sqrt{1+ \sqrt{1 + ...}}} ## is basically the same as ## x = \sqrt{1 + x}##
 
jedishrfu said:
I'd investigate nested radicals as this was a favorite of Ramanujan.

As a hint,## x = \sqrt{1 + \sqrt{1+ \sqrt{1 + ...}}} ## is basically the same as ## x = \sqrt{1 + x}##
That hasn't helped. Please explain.
 
Mathematicsss said:
(ai) = 1, sqrt3, sqrt(1+sqrt3), sqrt(1+sqrt(1+sqrt2))
Shouldn't the last one you listed be ##\sqrt{1 + \sqrt{1 + \sqrt 3}}##?
Start by listing the elements of your sequence in an organized fashion, like so:
##a_0 = 1##
##a_1 = \sqrt 3##
##a_2 = \sqrt{1 + \sqrt 3}## What is ##a_2## in terms of ##a_1##?
##a_3 = \sqrt{1 + \sqrt{1 + \sqrt 3}}## What is ##a_3## in terms of ##a_2##?
Can you predict what ##a_4## is? If you can, you might be able to write ##a_n## in terms of ##a_{n - 1}##, which is what you need to do for this problem.
 
Question: A clock's minute hand has length 4 and its hour hand has length 3. What is the distance between the tips at the moment when it is increasing most rapidly?(Putnam Exam Question) Answer: Making assumption that both the hands moves at constant angular velocities, the answer is ## \sqrt{7} .## But don't you think this assumption is somewhat doubtful and wrong?

Similar threads

  • · Replies 11 ·
Replies
11
Views
2K
  • · Replies 2 ·
Replies
2
Views
1K
Replies
3
Views
2K
  • · Replies 1 ·
Replies
1
Views
2K
Replies
2
Views
1K
  • · Replies 9 ·
Replies
9
Views
3K
  • · Replies 2 ·
Replies
2
Views
2K
  • · Replies 7 ·
Replies
7
Views
2K
Replies
1
Views
2K
  • · Replies 1 ·
Replies
1
Views
1K